Baaad: $int_0^{pi/2}logbigg(frac{2+sin2x}{2-sin2x}bigg)mathrm dx$












12












$begingroup$


I am in the process of proving
$$I=int_0^infty frac{arctan x}{x^4+x^2+1}mathrm{d}x=frac{pi^2}{8sqrt{3}}-frac23G+fracpi{12}log(2+sqrt{3})$$
And I have gotten as far as showing that
$$2I=frac{pi^2}{4sqrt{3}}+J$$
Where
$$J=int_0^infty logbigg(frac{x^2-x+1}{x^2+x+1}bigg)frac{mathrm{d}x}{1+x^2}$$
Then we preform $x=tan u$ to see that
$$J=int_0^{pi/2}logbigg(frac{2+sin2x}{2-sin2x}bigg)mathrm dx$$
Which I have been stuck on for the past while. I tried defining
$$k(a)=int_0^{pi/2}log(2+sin2ax)mathrm dx$$
Which gives
$$J=k(1)-k(-1)$$
Then differentiating under the integral:
$$k'(a)=2int_0^{pi/2}frac{xcos2ax}{2+sin2ax}mathrm dx$$
We may integrate by parts with $u=x$ to get a differential equation
$$ak'(a)+k(a)=fracpi2log(2+sinpi a)$$
With initial condition
$$k(0)=fracpi2log2$$
And from here I have no idea what to do.



I also tried tangent half angle substitution, but that just gave me the original expression for $J$.



I'm hoping that there is some really easy method that just never occurred to me... Any tips?



Edit



As was pointed out in the comments, I could consider
$$P(a)=frac12int_0^pi log(a+sin x)mathrm dx\Rightarrow P(0)=-fracpi2log2$$
And
$$
begin{align}
Q(a)=&frac12int_0^pi log(a-sin x)mathrm dx\
=&frac12int_0^pilog[-(-a+sin x)]mathrm dx\
=&frac12int_0^pibigg(log(-1)+log(-a+sin x)bigg)mathrm dx\
=&frac{ipi}2int_0^pimathrm{d}x+frac12int_0^pilog(-a+sin x)mathrm dx\
=&frac{ipi^2}2+P(-a)
end{align}
$$

Hence
$$J=P(2)-Q(2)=P(2)-P(-2)-frac{ipi^2}2$$
So now we care about $P(a)$. Differentiating under the integral, we have
$$P'(a)=frac12int_0^pi frac{mathrm{d}x}{a+sin x}$$
With a healthy dose of Tangent half angle substitution,
$$P'(a)=int_0^infty frac{mathrm{d}x}{ax^2+2x+a}$$
completing the square, we have
$$P'(a)=int_0^infty frac{mathrm{d}x}{a(x+frac1a)^2+g}$$
Where $g=a-frac1a$. With the right trigonometric substitution,
$$P'(a)=frac1{sqrt{a^2+1}}int_{x_1}^{pi/2}mathrm{d}x$$
Where $x_1=arctanfrac1{sqrt{a^2+1}}$. Then using
$$arctanfrac1x=fracpi2-arctan x$$
We have that
$$P'(a)=frac1{sqrt{a^2+1}}arctansqrt{a^2+1}$$
So we end up with something I don't know how to to deal with (what a surprise)
$$P(a)=intarctansqrt{a^2+1}frac{mathrm{d}a}{sqrt{a^2+1}}$$
Could you help me out with this last one? Thanks.










share|cite|improve this question











$endgroup$








  • 1




    $begingroup$
    So $(ak(a))'=frac{pi}{2}log{(2+sin{pi a})}$?
    $endgroup$
    – Lau
    Dec 18 '18 at 2:25










  • $begingroup$
    @Lau Oh... didn't think of that...
    $endgroup$
    – clathratus
    Dec 18 '18 at 2:30










  • $begingroup$
    But I think it may not do any help ... It seems that you will go back to $k(a)$.
    $endgroup$
    – Lau
    Dec 18 '18 at 2:37










  • $begingroup$
    @Lau yeah, $$ak(a)=fracpi2intlog(2+sinpi a)mathrm{d}a$$ which is fishily similar to $k(a)$.
    $endgroup$
    – clathratus
    Dec 18 '18 at 2:39






  • 1




    $begingroup$
    Here's an idea: when defining $k(a)$, take $log(a+sin(2x))$ instead.
    $endgroup$
    – Nastar
    Dec 18 '18 at 3:17
















12












$begingroup$


I am in the process of proving
$$I=int_0^infty frac{arctan x}{x^4+x^2+1}mathrm{d}x=frac{pi^2}{8sqrt{3}}-frac23G+fracpi{12}log(2+sqrt{3})$$
And I have gotten as far as showing that
$$2I=frac{pi^2}{4sqrt{3}}+J$$
Where
$$J=int_0^infty logbigg(frac{x^2-x+1}{x^2+x+1}bigg)frac{mathrm{d}x}{1+x^2}$$
Then we preform $x=tan u$ to see that
$$J=int_0^{pi/2}logbigg(frac{2+sin2x}{2-sin2x}bigg)mathrm dx$$
Which I have been stuck on for the past while. I tried defining
$$k(a)=int_0^{pi/2}log(2+sin2ax)mathrm dx$$
Which gives
$$J=k(1)-k(-1)$$
Then differentiating under the integral:
$$k'(a)=2int_0^{pi/2}frac{xcos2ax}{2+sin2ax}mathrm dx$$
We may integrate by parts with $u=x$ to get a differential equation
$$ak'(a)+k(a)=fracpi2log(2+sinpi a)$$
With initial condition
$$k(0)=fracpi2log2$$
And from here I have no idea what to do.



I also tried tangent half angle substitution, but that just gave me the original expression for $J$.



I'm hoping that there is some really easy method that just never occurred to me... Any tips?



Edit



As was pointed out in the comments, I could consider
$$P(a)=frac12int_0^pi log(a+sin x)mathrm dx\Rightarrow P(0)=-fracpi2log2$$
And
$$
begin{align}
Q(a)=&frac12int_0^pi log(a-sin x)mathrm dx\
=&frac12int_0^pilog[-(-a+sin x)]mathrm dx\
=&frac12int_0^pibigg(log(-1)+log(-a+sin x)bigg)mathrm dx\
=&frac{ipi}2int_0^pimathrm{d}x+frac12int_0^pilog(-a+sin x)mathrm dx\
=&frac{ipi^2}2+P(-a)
end{align}
$$

Hence
$$J=P(2)-Q(2)=P(2)-P(-2)-frac{ipi^2}2$$
So now we care about $P(a)$. Differentiating under the integral, we have
$$P'(a)=frac12int_0^pi frac{mathrm{d}x}{a+sin x}$$
With a healthy dose of Tangent half angle substitution,
$$P'(a)=int_0^infty frac{mathrm{d}x}{ax^2+2x+a}$$
completing the square, we have
$$P'(a)=int_0^infty frac{mathrm{d}x}{a(x+frac1a)^2+g}$$
Where $g=a-frac1a$. With the right trigonometric substitution,
$$P'(a)=frac1{sqrt{a^2+1}}int_{x_1}^{pi/2}mathrm{d}x$$
Where $x_1=arctanfrac1{sqrt{a^2+1}}$. Then using
$$arctanfrac1x=fracpi2-arctan x$$
We have that
$$P'(a)=frac1{sqrt{a^2+1}}arctansqrt{a^2+1}$$
So we end up with something I don't know how to to deal with (what a surprise)
$$P(a)=intarctansqrt{a^2+1}frac{mathrm{d}a}{sqrt{a^2+1}}$$
Could you help me out with this last one? Thanks.










share|cite|improve this question











$endgroup$








  • 1




    $begingroup$
    So $(ak(a))'=frac{pi}{2}log{(2+sin{pi a})}$?
    $endgroup$
    – Lau
    Dec 18 '18 at 2:25










  • $begingroup$
    @Lau Oh... didn't think of that...
    $endgroup$
    – clathratus
    Dec 18 '18 at 2:30










  • $begingroup$
    But I think it may not do any help ... It seems that you will go back to $k(a)$.
    $endgroup$
    – Lau
    Dec 18 '18 at 2:37










  • $begingroup$
    @Lau yeah, $$ak(a)=fracpi2intlog(2+sinpi a)mathrm{d}a$$ which is fishily similar to $k(a)$.
    $endgroup$
    – clathratus
    Dec 18 '18 at 2:39






  • 1




    $begingroup$
    Here's an idea: when defining $k(a)$, take $log(a+sin(2x))$ instead.
    $endgroup$
    – Nastar
    Dec 18 '18 at 3:17














12












12








12


5



$begingroup$


I am in the process of proving
$$I=int_0^infty frac{arctan x}{x^4+x^2+1}mathrm{d}x=frac{pi^2}{8sqrt{3}}-frac23G+fracpi{12}log(2+sqrt{3})$$
And I have gotten as far as showing that
$$2I=frac{pi^2}{4sqrt{3}}+J$$
Where
$$J=int_0^infty logbigg(frac{x^2-x+1}{x^2+x+1}bigg)frac{mathrm{d}x}{1+x^2}$$
Then we preform $x=tan u$ to see that
$$J=int_0^{pi/2}logbigg(frac{2+sin2x}{2-sin2x}bigg)mathrm dx$$
Which I have been stuck on for the past while. I tried defining
$$k(a)=int_0^{pi/2}log(2+sin2ax)mathrm dx$$
Which gives
$$J=k(1)-k(-1)$$
Then differentiating under the integral:
$$k'(a)=2int_0^{pi/2}frac{xcos2ax}{2+sin2ax}mathrm dx$$
We may integrate by parts with $u=x$ to get a differential equation
$$ak'(a)+k(a)=fracpi2log(2+sinpi a)$$
With initial condition
$$k(0)=fracpi2log2$$
And from here I have no idea what to do.



I also tried tangent half angle substitution, but that just gave me the original expression for $J$.



I'm hoping that there is some really easy method that just never occurred to me... Any tips?



Edit



As was pointed out in the comments, I could consider
$$P(a)=frac12int_0^pi log(a+sin x)mathrm dx\Rightarrow P(0)=-fracpi2log2$$
And
$$
begin{align}
Q(a)=&frac12int_0^pi log(a-sin x)mathrm dx\
=&frac12int_0^pilog[-(-a+sin x)]mathrm dx\
=&frac12int_0^pibigg(log(-1)+log(-a+sin x)bigg)mathrm dx\
=&frac{ipi}2int_0^pimathrm{d}x+frac12int_0^pilog(-a+sin x)mathrm dx\
=&frac{ipi^2}2+P(-a)
end{align}
$$

Hence
$$J=P(2)-Q(2)=P(2)-P(-2)-frac{ipi^2}2$$
So now we care about $P(a)$. Differentiating under the integral, we have
$$P'(a)=frac12int_0^pi frac{mathrm{d}x}{a+sin x}$$
With a healthy dose of Tangent half angle substitution,
$$P'(a)=int_0^infty frac{mathrm{d}x}{ax^2+2x+a}$$
completing the square, we have
$$P'(a)=int_0^infty frac{mathrm{d}x}{a(x+frac1a)^2+g}$$
Where $g=a-frac1a$. With the right trigonometric substitution,
$$P'(a)=frac1{sqrt{a^2+1}}int_{x_1}^{pi/2}mathrm{d}x$$
Where $x_1=arctanfrac1{sqrt{a^2+1}}$. Then using
$$arctanfrac1x=fracpi2-arctan x$$
We have that
$$P'(a)=frac1{sqrt{a^2+1}}arctansqrt{a^2+1}$$
So we end up with something I don't know how to to deal with (what a surprise)
$$P(a)=intarctansqrt{a^2+1}frac{mathrm{d}a}{sqrt{a^2+1}}$$
Could you help me out with this last one? Thanks.










share|cite|improve this question











$endgroup$




I am in the process of proving
$$I=int_0^infty frac{arctan x}{x^4+x^2+1}mathrm{d}x=frac{pi^2}{8sqrt{3}}-frac23G+fracpi{12}log(2+sqrt{3})$$
And I have gotten as far as showing that
$$2I=frac{pi^2}{4sqrt{3}}+J$$
Where
$$J=int_0^infty logbigg(frac{x^2-x+1}{x^2+x+1}bigg)frac{mathrm{d}x}{1+x^2}$$
Then we preform $x=tan u$ to see that
$$J=int_0^{pi/2}logbigg(frac{2+sin2x}{2-sin2x}bigg)mathrm dx$$
Which I have been stuck on for the past while. I tried defining
$$k(a)=int_0^{pi/2}log(2+sin2ax)mathrm dx$$
Which gives
$$J=k(1)-k(-1)$$
Then differentiating under the integral:
$$k'(a)=2int_0^{pi/2}frac{xcos2ax}{2+sin2ax}mathrm dx$$
We may integrate by parts with $u=x$ to get a differential equation
$$ak'(a)+k(a)=fracpi2log(2+sinpi a)$$
With initial condition
$$k(0)=fracpi2log2$$
And from here I have no idea what to do.



I also tried tangent half angle substitution, but that just gave me the original expression for $J$.



I'm hoping that there is some really easy method that just never occurred to me... Any tips?



Edit



As was pointed out in the comments, I could consider
$$P(a)=frac12int_0^pi log(a+sin x)mathrm dx\Rightarrow P(0)=-fracpi2log2$$
And
$$
begin{align}
Q(a)=&frac12int_0^pi log(a-sin x)mathrm dx\
=&frac12int_0^pilog[-(-a+sin x)]mathrm dx\
=&frac12int_0^pibigg(log(-1)+log(-a+sin x)bigg)mathrm dx\
=&frac{ipi}2int_0^pimathrm{d}x+frac12int_0^pilog(-a+sin x)mathrm dx\
=&frac{ipi^2}2+P(-a)
end{align}
$$

Hence
$$J=P(2)-Q(2)=P(2)-P(-2)-frac{ipi^2}2$$
So now we care about $P(a)$. Differentiating under the integral, we have
$$P'(a)=frac12int_0^pi frac{mathrm{d}x}{a+sin x}$$
With a healthy dose of Tangent half angle substitution,
$$P'(a)=int_0^infty frac{mathrm{d}x}{ax^2+2x+a}$$
completing the square, we have
$$P'(a)=int_0^infty frac{mathrm{d}x}{a(x+frac1a)^2+g}$$
Where $g=a-frac1a$. With the right trigonometric substitution,
$$P'(a)=frac1{sqrt{a^2+1}}int_{x_1}^{pi/2}mathrm{d}x$$
Where $x_1=arctanfrac1{sqrt{a^2+1}}$. Then using
$$arctanfrac1x=fracpi2-arctan x$$
We have that
$$P'(a)=frac1{sqrt{a^2+1}}arctansqrt{a^2+1}$$
So we end up with something I don't know how to to deal with (what a surprise)
$$P(a)=intarctansqrt{a^2+1}frac{mathrm{d}a}{sqrt{a^2+1}}$$
Could you help me out with this last one? Thanks.







integration closed-form






share|cite|improve this question















share|cite|improve this question













share|cite|improve this question




share|cite|improve this question








edited Mar 3 at 1:43







clathratus

















asked Dec 18 '18 at 1:42









clathratusclathratus

5,0101338




5,0101338








  • 1




    $begingroup$
    So $(ak(a))'=frac{pi}{2}log{(2+sin{pi a})}$?
    $endgroup$
    – Lau
    Dec 18 '18 at 2:25










  • $begingroup$
    @Lau Oh... didn't think of that...
    $endgroup$
    – clathratus
    Dec 18 '18 at 2:30










  • $begingroup$
    But I think it may not do any help ... It seems that you will go back to $k(a)$.
    $endgroup$
    – Lau
    Dec 18 '18 at 2:37










  • $begingroup$
    @Lau yeah, $$ak(a)=fracpi2intlog(2+sinpi a)mathrm{d}a$$ which is fishily similar to $k(a)$.
    $endgroup$
    – clathratus
    Dec 18 '18 at 2:39






  • 1




    $begingroup$
    Here's an idea: when defining $k(a)$, take $log(a+sin(2x))$ instead.
    $endgroup$
    – Nastar
    Dec 18 '18 at 3:17














  • 1




    $begingroup$
    So $(ak(a))'=frac{pi}{2}log{(2+sin{pi a})}$?
    $endgroup$
    – Lau
    Dec 18 '18 at 2:25










  • $begingroup$
    @Lau Oh... didn't think of that...
    $endgroup$
    – clathratus
    Dec 18 '18 at 2:30










  • $begingroup$
    But I think it may not do any help ... It seems that you will go back to $k(a)$.
    $endgroup$
    – Lau
    Dec 18 '18 at 2:37










  • $begingroup$
    @Lau yeah, $$ak(a)=fracpi2intlog(2+sinpi a)mathrm{d}a$$ which is fishily similar to $k(a)$.
    $endgroup$
    – clathratus
    Dec 18 '18 at 2:39






  • 1




    $begingroup$
    Here's an idea: when defining $k(a)$, take $log(a+sin(2x))$ instead.
    $endgroup$
    – Nastar
    Dec 18 '18 at 3:17








1




1




$begingroup$
So $(ak(a))'=frac{pi}{2}log{(2+sin{pi a})}$?
$endgroup$
– Lau
Dec 18 '18 at 2:25




$begingroup$
So $(ak(a))'=frac{pi}{2}log{(2+sin{pi a})}$?
$endgroup$
– Lau
Dec 18 '18 at 2:25












$begingroup$
@Lau Oh... didn't think of that...
$endgroup$
– clathratus
Dec 18 '18 at 2:30




$begingroup$
@Lau Oh... didn't think of that...
$endgroup$
– clathratus
Dec 18 '18 at 2:30












$begingroup$
But I think it may not do any help ... It seems that you will go back to $k(a)$.
$endgroup$
– Lau
Dec 18 '18 at 2:37




$begingroup$
But I think it may not do any help ... It seems that you will go back to $k(a)$.
$endgroup$
– Lau
Dec 18 '18 at 2:37












$begingroup$
@Lau yeah, $$ak(a)=fracpi2intlog(2+sinpi a)mathrm{d}a$$ which is fishily similar to $k(a)$.
$endgroup$
– clathratus
Dec 18 '18 at 2:39




$begingroup$
@Lau yeah, $$ak(a)=fracpi2intlog(2+sinpi a)mathrm{d}a$$ which is fishily similar to $k(a)$.
$endgroup$
– clathratus
Dec 18 '18 at 2:39




1




1




$begingroup$
Here's an idea: when defining $k(a)$, take $log(a+sin(2x))$ instead.
$endgroup$
– Nastar
Dec 18 '18 at 3:17




$begingroup$
Here's an idea: when defining $k(a)$, take $log(a+sin(2x))$ instead.
$endgroup$
– Nastar
Dec 18 '18 at 3:17










3 Answers
3






active

oldest

votes


















7












$begingroup$

$$J=int_0^{pi/2}lnleft(frac{2+sin2x}{2-sin2x}right)mathrm dxoverset{2x=t}=frac12 int_0^pi lnleft(frac{1+frac12sin t}{1-frac12sin t}right)mathrm dt=int_0^frac{pi}{2}lnleft(frac{1+frac12sin x}{1-frac12sin x }right)mathrm dx$$
$${lnleft(frac{1+ysin x}{1-ysin x}right)+c=2intfrac{sin x}{1-y^2sin^2x}dy}Rightarrowlnleft(frac{1+frac12sin t}{1-frac12sin t }right)=2int_0^frac{1}{2}frac{sin x}{1-y^2sin^2x}dy tag1$$



By Fubini's theorem we may interchange the order of the integrals and using $(1)$ we get: $$J=2int_0^frac{pi}{2}int_0^frac12 frac{sin x}{1-y^2sin^2x}dydx=2int_0^frac12int_0^frac{pi}{2} frac{sin x}{1-y^2sin^2x}dxdy=$$
$$=2int_0^frac12int_0^frac{pi}{2} frac{sin x}{y^2cos^2x +1-y^2} dxdy=2int_0^frac12frac{1}{ysqrt{1-y^2}}arctanleft(frac{yx}{sqrt{1-y^2}}right)bigg|_0^1 dy=$$
$$=2int_0^frac12frac{1}{ysqrt{1-y^2}}arctanleft(frac{y}{sqrt{1-y^2}}right)dyoverset{y=sin x}=2int_0^frac{pi}{6}frac{x}{sin x}dx=$$
$$=2int_0^{frac{pi}{6}} x left(lnleft(tan frac{x}{2}right)right)'dx=2x lnleft(tan frac{x}{2}right)bigg|_0^{frac{pi}{6}} -2{int_0^{frac{pi}{6}} lnleft(tan frac{x}{2}right)dx}=$$
$$overset{frac{x}{2}=t}=frac{pi}{3}ln(2-sqrt 3) -4int_0^frac{pi}{12}ln (tan t)dt=-frac{pi}{3}ln(2+sqrt 3) +frac{8}{3}G$$ $G$ is the Catalan's constant and for the last integral see here.





Also note that you should have: $$2I=frac{pi^2}{4sqrt 3}- int_0^inftyfrac{(x^2-1)arctan x}{x^4+x^2+1}dx=frac{pi^2}{4sqrt 3}-frac12underset{=J}{int_0^infty lnbigg(frac{x^2-x+1}{x^2+x+1}bigg)frac{dx}{1+x^2}}$$





As an alternative employ Feynman's trick for $,displaystyle{I(a)=int_0^frac{pi}{2}lnleft(frac{1+asin x}{1-asin x}right)dt},$ to get:
$$I'(a)=int_0^frac{pi}{2} left(frac{sin x}{1+asin x}+frac{sin x}{1-asin x}right)dx =2int_0^frac{pi}{2} frac{sin x}{1-a^2sin^2 x}dx=$$
$$=2int_0^frac{pi}{2} frac{sin x}{a^2cos^2 x + left(sqrt{1-a^2}right)^2}dx=frac{2}{a^2}int_0^1 frac{d(cos x)}{cos^2x +left(frac{sqrt{1-a^2}}{a}right)^2}=$$
$$=frac{2a}{a^2sqrt{1-a^2}}arctanleft(frac{ax}{sqrt{1-a^2}}right)bigg|_0^1=frac{2}{asqrt{1-a^2}}arctanleft(frac{a}{sqrt{1-a^2}}right)$$
$$I(0)=0 Rightarrow J=I(tfrac12)=int_0^frac12 frac{2}{asqrt{1-a^2}}arctanleft(frac{a}{sqrt{1-a^2}}right)overset{a=sin x}=2int_0^frac{pi}{6}frac{x}{sin x}dx$$
And the rest part is found above.






share|cite|improve this answer











$endgroup$









  • 1




    $begingroup$
    Shouldn't you give justification for differentiating under the integral?
    $endgroup$
    – user21820
    Dec 18 '18 at 9:23






  • 3




    $begingroup$
    That is just Feynman's trick, or differentiating under the integral sign. I don't understand what I should justify at all. See here for example for more: medium.com/dialogue-and-discourse/… and here: brilliant.org/wiki/integration-tricks/…
    $endgroup$
    – Zacky
    Dec 18 '18 at 9:37












  • $begingroup$
    It is not always valid, and the fact that you don't understand the precise conditions under which you can use it means that you shouldn't be using it! Both your sources are mathematically deficit for the same reason.
    $endgroup$
    – user21820
    Dec 18 '18 at 9:49








  • 2




    $begingroup$
    Sure. Please read this pdf by Keith Conrad for counter-examples and a full mathematical explanation.
    $endgroup$
    – user21820
    Dec 18 '18 at 9:56






  • 2




    $begingroup$
    Legit!! What a neat solution! Thanks Zacky!
    $endgroup$
    – clathratus
    Dec 18 '18 at 19:04



















3












$begingroup$

A Possible way: Consider
$$I(a)=int_{0}^{+infty}frac{arctan(ax)}{1+x^2+x^4} dx$$
and
$$I'(a)=int_{0}^{+infty}frac{x}{(1+x^2+x^4)(1+x^2a^2)}dx=int_{0}^{+infty}frac{1}{(1+y+y^2)(1+a^2y)}dx$$
and



$$frac{1}{(1+y+y^2)(1+a^2y)}= frac{-a^2y-a^2+1}{(a^4-a^2+1)(1+y+y^2)}+frac{a^4}{(a^4-a^2+1)(ay^2+1)}$$



we can also calculate $I'(a)$ by complex integration (if you've learned that).



Thanks to Dylan for his advice.






share|cite|improve this answer











$endgroup$













  • $begingroup$
    Instead of doing partial fractions wrt $x$, you might consider substituting $u=x^2$ first
    $endgroup$
    – Dylan
    Dec 18 '18 at 3:07












  • $begingroup$
    And it is still not so easy. I think there may be a more clever way to solve it.
    $endgroup$
    – Lau
    Dec 18 '18 at 3:15










  • $begingroup$
    I meant the first integration. $$frac12 int_0^infty frac{1}{(1+u+u^2)(1+a^2u)} du$$ and then do partial fractions?
    $endgroup$
    – Dylan
    Dec 18 '18 at 3:31












  • $begingroup$
    @Dylan You're right.
    $endgroup$
    – Lau
    Dec 18 '18 at 3:34








  • 1




    $begingroup$
    If you have to know complex analysis in order to do complex integration, then I can't do it. If complex integration is just normal integration but with a few $i$'s here and there, then I can do that.
    $endgroup$
    – clathratus
    Dec 18 '18 at 5:03



















3












$begingroup$

Result



I find that the integral has a closed form given by



$$i = intlimits_0^{pi/2}logbigg(frac{2+sin2x}{2-sin2x}bigg)mathrm dx = frac{1}{3} left(8 C-pi log left(2+sqrt{3}right)right) simeq 1.06346tag{1}$$



where



$$C = sum _{k=1}^{infty } frac{(-1)^{k+1}}{(2 k-1)^2} simeq 0.915966$$



is Catalan's constant.



Heuristic derivation



Notice trivially that because of the symmtery of the integrand the integral can be written as twice the integral from $0$ to $frac{pi}{4}$ which we shall utilize in what follows.



The basic idea is the series expansion



$$log left(frac{1+z}{1-z}right)=2tanh ^{-1}(z) = 2 sum _{k=1}^{infty } frac{z^{2 k-1}}{2 k-1},|z|<1 tag{2}$$



The integral is then to be done over the odd powers of the $sin$ with the result



$$int_0^{frac{pi }{4}} sin ^{2 k-1}(2 x) , dx = frac{sqrt{pi } Gamma (k)}{4 Gamma left(k+frac{1}{2}right)}tag{3}$$



Assembling the pieces the sum to be taken to represent $i$ becomes



$$i_s = sum _{k=1}^{infty } frac{sqrt{pi } Gamma (k)}{(2 k-1) 2^{2 k-1} Gamma left(k+frac{1}{2}right)}tag{4}$$



and this sum is immediately computed by Mathematica to give the compact result $(1)$.



Let us make the sum more transparent using the chain



$$frac{sqrt{pi } Gamma (k)}{Gamma left(k+frac{1}{2}right)}=Bleft(frac{1}{2},kright)=int_0^1 frac{t^{k-1}}{sqrt{1-t}} , dttag{5}$$



and doing the sum under the integral



$$sum _{k=1}^{infty } frac{t^{k-1}}{(2 k-1) 2^{2 k-1}}=frac{tanh ^{-1}left(frac{sqrt{t}}{2}right)}{sqrt{t}}tag{6}$$



leads finally to the integral



$$int_0^1 frac{tanh ^{-1}left(frac{sqrt{t}}{2}right)}{sqrt{t} sqrt{1-t}} , dttag{7}$$



for which Mathematica again quickly gives (1).



But there must be a shorter way ... yes, it is, substituting $sin (2 x)=sqrt{t}$ in the original integral gives (7) directly.






share|cite|improve this answer











$endgroup$













  • $begingroup$
    I do truly work better with series. Thank you.
    $endgroup$
    – clathratus
    Dec 18 '18 at 18:22











Your Answer





StackExchange.ifUsing("editor", function () {
return StackExchange.using("mathjaxEditing", function () {
StackExchange.MarkdownEditor.creationCallbacks.add(function (editor, postfix) {
StackExchange.mathjaxEditing.prepareWmdForMathJax(editor, postfix, [["$", "$"], ["\\(","\\)"]]);
});
});
}, "mathjax-editing");

StackExchange.ready(function() {
var channelOptions = {
tags: "".split(" "),
id: "69"
};
initTagRenderer("".split(" "), "".split(" "), channelOptions);

StackExchange.using("externalEditor", function() {
// Have to fire editor after snippets, if snippets enabled
if (StackExchange.settings.snippets.snippetsEnabled) {
StackExchange.using("snippets", function() {
createEditor();
});
}
else {
createEditor();
}
});

function createEditor() {
StackExchange.prepareEditor({
heartbeatType: 'answer',
autoActivateHeartbeat: false,
convertImagesToLinks: true,
noModals: true,
showLowRepImageUploadWarning: true,
reputationToPostImages: 10,
bindNavPrevention: true,
postfix: "",
imageUploader: {
brandingHtml: "Powered by u003ca class="icon-imgur-white" href="https://imgur.com/"u003eu003c/au003e",
contentPolicyHtml: "User contributions licensed under u003ca href="https://creativecommons.org/licenses/by-sa/3.0/"u003ecc by-sa 3.0 with attribution requiredu003c/au003e u003ca href="https://stackoverflow.com/legal/content-policy"u003e(content policy)u003c/au003e",
allowUrls: true
},
noCode: true, onDemand: true,
discardSelector: ".discard-answer"
,immediatelyShowMarkdownHelp:true
});


}
});














draft saved

draft discarded


















StackExchange.ready(
function () {
StackExchange.openid.initPostLogin('.new-post-login', 'https%3a%2f%2fmath.stackexchange.com%2fquestions%2f3044687%2fbaaad-int-0-pi-2-log-bigg-frac2-sin2x2-sin2x-bigg-mathrm-dx%23new-answer', 'question_page');
}
);

Post as a guest















Required, but never shown

























3 Answers
3






active

oldest

votes








3 Answers
3






active

oldest

votes









active

oldest

votes






active

oldest

votes









7












$begingroup$

$$J=int_0^{pi/2}lnleft(frac{2+sin2x}{2-sin2x}right)mathrm dxoverset{2x=t}=frac12 int_0^pi lnleft(frac{1+frac12sin t}{1-frac12sin t}right)mathrm dt=int_0^frac{pi}{2}lnleft(frac{1+frac12sin x}{1-frac12sin x }right)mathrm dx$$
$${lnleft(frac{1+ysin x}{1-ysin x}right)+c=2intfrac{sin x}{1-y^2sin^2x}dy}Rightarrowlnleft(frac{1+frac12sin t}{1-frac12sin t }right)=2int_0^frac{1}{2}frac{sin x}{1-y^2sin^2x}dy tag1$$



By Fubini's theorem we may interchange the order of the integrals and using $(1)$ we get: $$J=2int_0^frac{pi}{2}int_0^frac12 frac{sin x}{1-y^2sin^2x}dydx=2int_0^frac12int_0^frac{pi}{2} frac{sin x}{1-y^2sin^2x}dxdy=$$
$$=2int_0^frac12int_0^frac{pi}{2} frac{sin x}{y^2cos^2x +1-y^2} dxdy=2int_0^frac12frac{1}{ysqrt{1-y^2}}arctanleft(frac{yx}{sqrt{1-y^2}}right)bigg|_0^1 dy=$$
$$=2int_0^frac12frac{1}{ysqrt{1-y^2}}arctanleft(frac{y}{sqrt{1-y^2}}right)dyoverset{y=sin x}=2int_0^frac{pi}{6}frac{x}{sin x}dx=$$
$$=2int_0^{frac{pi}{6}} x left(lnleft(tan frac{x}{2}right)right)'dx=2x lnleft(tan frac{x}{2}right)bigg|_0^{frac{pi}{6}} -2{int_0^{frac{pi}{6}} lnleft(tan frac{x}{2}right)dx}=$$
$$overset{frac{x}{2}=t}=frac{pi}{3}ln(2-sqrt 3) -4int_0^frac{pi}{12}ln (tan t)dt=-frac{pi}{3}ln(2+sqrt 3) +frac{8}{3}G$$ $G$ is the Catalan's constant and for the last integral see here.





Also note that you should have: $$2I=frac{pi^2}{4sqrt 3}- int_0^inftyfrac{(x^2-1)arctan x}{x^4+x^2+1}dx=frac{pi^2}{4sqrt 3}-frac12underset{=J}{int_0^infty lnbigg(frac{x^2-x+1}{x^2+x+1}bigg)frac{dx}{1+x^2}}$$





As an alternative employ Feynman's trick for $,displaystyle{I(a)=int_0^frac{pi}{2}lnleft(frac{1+asin x}{1-asin x}right)dt},$ to get:
$$I'(a)=int_0^frac{pi}{2} left(frac{sin x}{1+asin x}+frac{sin x}{1-asin x}right)dx =2int_0^frac{pi}{2} frac{sin x}{1-a^2sin^2 x}dx=$$
$$=2int_0^frac{pi}{2} frac{sin x}{a^2cos^2 x + left(sqrt{1-a^2}right)^2}dx=frac{2}{a^2}int_0^1 frac{d(cos x)}{cos^2x +left(frac{sqrt{1-a^2}}{a}right)^2}=$$
$$=frac{2a}{a^2sqrt{1-a^2}}arctanleft(frac{ax}{sqrt{1-a^2}}right)bigg|_0^1=frac{2}{asqrt{1-a^2}}arctanleft(frac{a}{sqrt{1-a^2}}right)$$
$$I(0)=0 Rightarrow J=I(tfrac12)=int_0^frac12 frac{2}{asqrt{1-a^2}}arctanleft(frac{a}{sqrt{1-a^2}}right)overset{a=sin x}=2int_0^frac{pi}{6}frac{x}{sin x}dx$$
And the rest part is found above.






share|cite|improve this answer











$endgroup$









  • 1




    $begingroup$
    Shouldn't you give justification for differentiating under the integral?
    $endgroup$
    – user21820
    Dec 18 '18 at 9:23






  • 3




    $begingroup$
    That is just Feynman's trick, or differentiating under the integral sign. I don't understand what I should justify at all. See here for example for more: medium.com/dialogue-and-discourse/… and here: brilliant.org/wiki/integration-tricks/…
    $endgroup$
    – Zacky
    Dec 18 '18 at 9:37












  • $begingroup$
    It is not always valid, and the fact that you don't understand the precise conditions under which you can use it means that you shouldn't be using it! Both your sources are mathematically deficit for the same reason.
    $endgroup$
    – user21820
    Dec 18 '18 at 9:49








  • 2




    $begingroup$
    Sure. Please read this pdf by Keith Conrad for counter-examples and a full mathematical explanation.
    $endgroup$
    – user21820
    Dec 18 '18 at 9:56






  • 2




    $begingroup$
    Legit!! What a neat solution! Thanks Zacky!
    $endgroup$
    – clathratus
    Dec 18 '18 at 19:04
















7












$begingroup$

$$J=int_0^{pi/2}lnleft(frac{2+sin2x}{2-sin2x}right)mathrm dxoverset{2x=t}=frac12 int_0^pi lnleft(frac{1+frac12sin t}{1-frac12sin t}right)mathrm dt=int_0^frac{pi}{2}lnleft(frac{1+frac12sin x}{1-frac12sin x }right)mathrm dx$$
$${lnleft(frac{1+ysin x}{1-ysin x}right)+c=2intfrac{sin x}{1-y^2sin^2x}dy}Rightarrowlnleft(frac{1+frac12sin t}{1-frac12sin t }right)=2int_0^frac{1}{2}frac{sin x}{1-y^2sin^2x}dy tag1$$



By Fubini's theorem we may interchange the order of the integrals and using $(1)$ we get: $$J=2int_0^frac{pi}{2}int_0^frac12 frac{sin x}{1-y^2sin^2x}dydx=2int_0^frac12int_0^frac{pi}{2} frac{sin x}{1-y^2sin^2x}dxdy=$$
$$=2int_0^frac12int_0^frac{pi}{2} frac{sin x}{y^2cos^2x +1-y^2} dxdy=2int_0^frac12frac{1}{ysqrt{1-y^2}}arctanleft(frac{yx}{sqrt{1-y^2}}right)bigg|_0^1 dy=$$
$$=2int_0^frac12frac{1}{ysqrt{1-y^2}}arctanleft(frac{y}{sqrt{1-y^2}}right)dyoverset{y=sin x}=2int_0^frac{pi}{6}frac{x}{sin x}dx=$$
$$=2int_0^{frac{pi}{6}} x left(lnleft(tan frac{x}{2}right)right)'dx=2x lnleft(tan frac{x}{2}right)bigg|_0^{frac{pi}{6}} -2{int_0^{frac{pi}{6}} lnleft(tan frac{x}{2}right)dx}=$$
$$overset{frac{x}{2}=t}=frac{pi}{3}ln(2-sqrt 3) -4int_0^frac{pi}{12}ln (tan t)dt=-frac{pi}{3}ln(2+sqrt 3) +frac{8}{3}G$$ $G$ is the Catalan's constant and for the last integral see here.





Also note that you should have: $$2I=frac{pi^2}{4sqrt 3}- int_0^inftyfrac{(x^2-1)arctan x}{x^4+x^2+1}dx=frac{pi^2}{4sqrt 3}-frac12underset{=J}{int_0^infty lnbigg(frac{x^2-x+1}{x^2+x+1}bigg)frac{dx}{1+x^2}}$$





As an alternative employ Feynman's trick for $,displaystyle{I(a)=int_0^frac{pi}{2}lnleft(frac{1+asin x}{1-asin x}right)dt},$ to get:
$$I'(a)=int_0^frac{pi}{2} left(frac{sin x}{1+asin x}+frac{sin x}{1-asin x}right)dx =2int_0^frac{pi}{2} frac{sin x}{1-a^2sin^2 x}dx=$$
$$=2int_0^frac{pi}{2} frac{sin x}{a^2cos^2 x + left(sqrt{1-a^2}right)^2}dx=frac{2}{a^2}int_0^1 frac{d(cos x)}{cos^2x +left(frac{sqrt{1-a^2}}{a}right)^2}=$$
$$=frac{2a}{a^2sqrt{1-a^2}}arctanleft(frac{ax}{sqrt{1-a^2}}right)bigg|_0^1=frac{2}{asqrt{1-a^2}}arctanleft(frac{a}{sqrt{1-a^2}}right)$$
$$I(0)=0 Rightarrow J=I(tfrac12)=int_0^frac12 frac{2}{asqrt{1-a^2}}arctanleft(frac{a}{sqrt{1-a^2}}right)overset{a=sin x}=2int_0^frac{pi}{6}frac{x}{sin x}dx$$
And the rest part is found above.






share|cite|improve this answer











$endgroup$









  • 1




    $begingroup$
    Shouldn't you give justification for differentiating under the integral?
    $endgroup$
    – user21820
    Dec 18 '18 at 9:23






  • 3




    $begingroup$
    That is just Feynman's trick, or differentiating under the integral sign. I don't understand what I should justify at all. See here for example for more: medium.com/dialogue-and-discourse/… and here: brilliant.org/wiki/integration-tricks/…
    $endgroup$
    – Zacky
    Dec 18 '18 at 9:37












  • $begingroup$
    It is not always valid, and the fact that you don't understand the precise conditions under which you can use it means that you shouldn't be using it! Both your sources are mathematically deficit for the same reason.
    $endgroup$
    – user21820
    Dec 18 '18 at 9:49








  • 2




    $begingroup$
    Sure. Please read this pdf by Keith Conrad for counter-examples and a full mathematical explanation.
    $endgroup$
    – user21820
    Dec 18 '18 at 9:56






  • 2




    $begingroup$
    Legit!! What a neat solution! Thanks Zacky!
    $endgroup$
    – clathratus
    Dec 18 '18 at 19:04














7












7








7





$begingroup$

$$J=int_0^{pi/2}lnleft(frac{2+sin2x}{2-sin2x}right)mathrm dxoverset{2x=t}=frac12 int_0^pi lnleft(frac{1+frac12sin t}{1-frac12sin t}right)mathrm dt=int_0^frac{pi}{2}lnleft(frac{1+frac12sin x}{1-frac12sin x }right)mathrm dx$$
$${lnleft(frac{1+ysin x}{1-ysin x}right)+c=2intfrac{sin x}{1-y^2sin^2x}dy}Rightarrowlnleft(frac{1+frac12sin t}{1-frac12sin t }right)=2int_0^frac{1}{2}frac{sin x}{1-y^2sin^2x}dy tag1$$



By Fubini's theorem we may interchange the order of the integrals and using $(1)$ we get: $$J=2int_0^frac{pi}{2}int_0^frac12 frac{sin x}{1-y^2sin^2x}dydx=2int_0^frac12int_0^frac{pi}{2} frac{sin x}{1-y^2sin^2x}dxdy=$$
$$=2int_0^frac12int_0^frac{pi}{2} frac{sin x}{y^2cos^2x +1-y^2} dxdy=2int_0^frac12frac{1}{ysqrt{1-y^2}}arctanleft(frac{yx}{sqrt{1-y^2}}right)bigg|_0^1 dy=$$
$$=2int_0^frac12frac{1}{ysqrt{1-y^2}}arctanleft(frac{y}{sqrt{1-y^2}}right)dyoverset{y=sin x}=2int_0^frac{pi}{6}frac{x}{sin x}dx=$$
$$=2int_0^{frac{pi}{6}} x left(lnleft(tan frac{x}{2}right)right)'dx=2x lnleft(tan frac{x}{2}right)bigg|_0^{frac{pi}{6}} -2{int_0^{frac{pi}{6}} lnleft(tan frac{x}{2}right)dx}=$$
$$overset{frac{x}{2}=t}=frac{pi}{3}ln(2-sqrt 3) -4int_0^frac{pi}{12}ln (tan t)dt=-frac{pi}{3}ln(2+sqrt 3) +frac{8}{3}G$$ $G$ is the Catalan's constant and for the last integral see here.





Also note that you should have: $$2I=frac{pi^2}{4sqrt 3}- int_0^inftyfrac{(x^2-1)arctan x}{x^4+x^2+1}dx=frac{pi^2}{4sqrt 3}-frac12underset{=J}{int_0^infty lnbigg(frac{x^2-x+1}{x^2+x+1}bigg)frac{dx}{1+x^2}}$$





As an alternative employ Feynman's trick for $,displaystyle{I(a)=int_0^frac{pi}{2}lnleft(frac{1+asin x}{1-asin x}right)dt},$ to get:
$$I'(a)=int_0^frac{pi}{2} left(frac{sin x}{1+asin x}+frac{sin x}{1-asin x}right)dx =2int_0^frac{pi}{2} frac{sin x}{1-a^2sin^2 x}dx=$$
$$=2int_0^frac{pi}{2} frac{sin x}{a^2cos^2 x + left(sqrt{1-a^2}right)^2}dx=frac{2}{a^2}int_0^1 frac{d(cos x)}{cos^2x +left(frac{sqrt{1-a^2}}{a}right)^2}=$$
$$=frac{2a}{a^2sqrt{1-a^2}}arctanleft(frac{ax}{sqrt{1-a^2}}right)bigg|_0^1=frac{2}{asqrt{1-a^2}}arctanleft(frac{a}{sqrt{1-a^2}}right)$$
$$I(0)=0 Rightarrow J=I(tfrac12)=int_0^frac12 frac{2}{asqrt{1-a^2}}arctanleft(frac{a}{sqrt{1-a^2}}right)overset{a=sin x}=2int_0^frac{pi}{6}frac{x}{sin x}dx$$
And the rest part is found above.






share|cite|improve this answer











$endgroup$



$$J=int_0^{pi/2}lnleft(frac{2+sin2x}{2-sin2x}right)mathrm dxoverset{2x=t}=frac12 int_0^pi lnleft(frac{1+frac12sin t}{1-frac12sin t}right)mathrm dt=int_0^frac{pi}{2}lnleft(frac{1+frac12sin x}{1-frac12sin x }right)mathrm dx$$
$${lnleft(frac{1+ysin x}{1-ysin x}right)+c=2intfrac{sin x}{1-y^2sin^2x}dy}Rightarrowlnleft(frac{1+frac12sin t}{1-frac12sin t }right)=2int_0^frac{1}{2}frac{sin x}{1-y^2sin^2x}dy tag1$$



By Fubini's theorem we may interchange the order of the integrals and using $(1)$ we get: $$J=2int_0^frac{pi}{2}int_0^frac12 frac{sin x}{1-y^2sin^2x}dydx=2int_0^frac12int_0^frac{pi}{2} frac{sin x}{1-y^2sin^2x}dxdy=$$
$$=2int_0^frac12int_0^frac{pi}{2} frac{sin x}{y^2cos^2x +1-y^2} dxdy=2int_0^frac12frac{1}{ysqrt{1-y^2}}arctanleft(frac{yx}{sqrt{1-y^2}}right)bigg|_0^1 dy=$$
$$=2int_0^frac12frac{1}{ysqrt{1-y^2}}arctanleft(frac{y}{sqrt{1-y^2}}right)dyoverset{y=sin x}=2int_0^frac{pi}{6}frac{x}{sin x}dx=$$
$$=2int_0^{frac{pi}{6}} x left(lnleft(tan frac{x}{2}right)right)'dx=2x lnleft(tan frac{x}{2}right)bigg|_0^{frac{pi}{6}} -2{int_0^{frac{pi}{6}} lnleft(tan frac{x}{2}right)dx}=$$
$$overset{frac{x}{2}=t}=frac{pi}{3}ln(2-sqrt 3) -4int_0^frac{pi}{12}ln (tan t)dt=-frac{pi}{3}ln(2+sqrt 3) +frac{8}{3}G$$ $G$ is the Catalan's constant and for the last integral see here.





Also note that you should have: $$2I=frac{pi^2}{4sqrt 3}- int_0^inftyfrac{(x^2-1)arctan x}{x^4+x^2+1}dx=frac{pi^2}{4sqrt 3}-frac12underset{=J}{int_0^infty lnbigg(frac{x^2-x+1}{x^2+x+1}bigg)frac{dx}{1+x^2}}$$





As an alternative employ Feynman's trick for $,displaystyle{I(a)=int_0^frac{pi}{2}lnleft(frac{1+asin x}{1-asin x}right)dt},$ to get:
$$I'(a)=int_0^frac{pi}{2} left(frac{sin x}{1+asin x}+frac{sin x}{1-asin x}right)dx =2int_0^frac{pi}{2} frac{sin x}{1-a^2sin^2 x}dx=$$
$$=2int_0^frac{pi}{2} frac{sin x}{a^2cos^2 x + left(sqrt{1-a^2}right)^2}dx=frac{2}{a^2}int_0^1 frac{d(cos x)}{cos^2x +left(frac{sqrt{1-a^2}}{a}right)^2}=$$
$$=frac{2a}{a^2sqrt{1-a^2}}arctanleft(frac{ax}{sqrt{1-a^2}}right)bigg|_0^1=frac{2}{asqrt{1-a^2}}arctanleft(frac{a}{sqrt{1-a^2}}right)$$
$$I(0)=0 Rightarrow J=I(tfrac12)=int_0^frac12 frac{2}{asqrt{1-a^2}}arctanleft(frac{a}{sqrt{1-a^2}}right)overset{a=sin x}=2int_0^frac{pi}{6}frac{x}{sin x}dx$$
And the rest part is found above.







share|cite|improve this answer














share|cite|improve this answer



share|cite|improve this answer








edited Dec 23 '18 at 19:30

























answered Dec 18 '18 at 8:05









ZackyZacky

7,76011061




7,76011061








  • 1




    $begingroup$
    Shouldn't you give justification for differentiating under the integral?
    $endgroup$
    – user21820
    Dec 18 '18 at 9:23






  • 3




    $begingroup$
    That is just Feynman's trick, or differentiating under the integral sign. I don't understand what I should justify at all. See here for example for more: medium.com/dialogue-and-discourse/… and here: brilliant.org/wiki/integration-tricks/…
    $endgroup$
    – Zacky
    Dec 18 '18 at 9:37












  • $begingroup$
    It is not always valid, and the fact that you don't understand the precise conditions under which you can use it means that you shouldn't be using it! Both your sources are mathematically deficit for the same reason.
    $endgroup$
    – user21820
    Dec 18 '18 at 9:49








  • 2




    $begingroup$
    Sure. Please read this pdf by Keith Conrad for counter-examples and a full mathematical explanation.
    $endgroup$
    – user21820
    Dec 18 '18 at 9:56






  • 2




    $begingroup$
    Legit!! What a neat solution! Thanks Zacky!
    $endgroup$
    – clathratus
    Dec 18 '18 at 19:04














  • 1




    $begingroup$
    Shouldn't you give justification for differentiating under the integral?
    $endgroup$
    – user21820
    Dec 18 '18 at 9:23






  • 3




    $begingroup$
    That is just Feynman's trick, or differentiating under the integral sign. I don't understand what I should justify at all. See here for example for more: medium.com/dialogue-and-discourse/… and here: brilliant.org/wiki/integration-tricks/…
    $endgroup$
    – Zacky
    Dec 18 '18 at 9:37












  • $begingroup$
    It is not always valid, and the fact that you don't understand the precise conditions under which you can use it means that you shouldn't be using it! Both your sources are mathematically deficit for the same reason.
    $endgroup$
    – user21820
    Dec 18 '18 at 9:49








  • 2




    $begingroup$
    Sure. Please read this pdf by Keith Conrad for counter-examples and a full mathematical explanation.
    $endgroup$
    – user21820
    Dec 18 '18 at 9:56






  • 2




    $begingroup$
    Legit!! What a neat solution! Thanks Zacky!
    $endgroup$
    – clathratus
    Dec 18 '18 at 19:04








1




1




$begingroup$
Shouldn't you give justification for differentiating under the integral?
$endgroup$
– user21820
Dec 18 '18 at 9:23




$begingroup$
Shouldn't you give justification for differentiating under the integral?
$endgroup$
– user21820
Dec 18 '18 at 9:23




3




3




$begingroup$
That is just Feynman's trick, or differentiating under the integral sign. I don't understand what I should justify at all. See here for example for more: medium.com/dialogue-and-discourse/… and here: brilliant.org/wiki/integration-tricks/…
$endgroup$
– Zacky
Dec 18 '18 at 9:37






$begingroup$
That is just Feynman's trick, or differentiating under the integral sign. I don't understand what I should justify at all. See here for example for more: medium.com/dialogue-and-discourse/… and here: brilliant.org/wiki/integration-tricks/…
$endgroup$
– Zacky
Dec 18 '18 at 9:37














$begingroup$
It is not always valid, and the fact that you don't understand the precise conditions under which you can use it means that you shouldn't be using it! Both your sources are mathematically deficit for the same reason.
$endgroup$
– user21820
Dec 18 '18 at 9:49






$begingroup$
It is not always valid, and the fact that you don't understand the precise conditions under which you can use it means that you shouldn't be using it! Both your sources are mathematically deficit for the same reason.
$endgroup$
– user21820
Dec 18 '18 at 9:49






2




2




$begingroup$
Sure. Please read this pdf by Keith Conrad for counter-examples and a full mathematical explanation.
$endgroup$
– user21820
Dec 18 '18 at 9:56




$begingroup$
Sure. Please read this pdf by Keith Conrad for counter-examples and a full mathematical explanation.
$endgroup$
– user21820
Dec 18 '18 at 9:56




2




2




$begingroup$
Legit!! What a neat solution! Thanks Zacky!
$endgroup$
– clathratus
Dec 18 '18 at 19:04




$begingroup$
Legit!! What a neat solution! Thanks Zacky!
$endgroup$
– clathratus
Dec 18 '18 at 19:04











3












$begingroup$

A Possible way: Consider
$$I(a)=int_{0}^{+infty}frac{arctan(ax)}{1+x^2+x^4} dx$$
and
$$I'(a)=int_{0}^{+infty}frac{x}{(1+x^2+x^4)(1+x^2a^2)}dx=int_{0}^{+infty}frac{1}{(1+y+y^2)(1+a^2y)}dx$$
and



$$frac{1}{(1+y+y^2)(1+a^2y)}= frac{-a^2y-a^2+1}{(a^4-a^2+1)(1+y+y^2)}+frac{a^4}{(a^4-a^2+1)(ay^2+1)}$$



we can also calculate $I'(a)$ by complex integration (if you've learned that).



Thanks to Dylan for his advice.






share|cite|improve this answer











$endgroup$













  • $begingroup$
    Instead of doing partial fractions wrt $x$, you might consider substituting $u=x^2$ first
    $endgroup$
    – Dylan
    Dec 18 '18 at 3:07












  • $begingroup$
    And it is still not so easy. I think there may be a more clever way to solve it.
    $endgroup$
    – Lau
    Dec 18 '18 at 3:15










  • $begingroup$
    I meant the first integration. $$frac12 int_0^infty frac{1}{(1+u+u^2)(1+a^2u)} du$$ and then do partial fractions?
    $endgroup$
    – Dylan
    Dec 18 '18 at 3:31












  • $begingroup$
    @Dylan You're right.
    $endgroup$
    – Lau
    Dec 18 '18 at 3:34








  • 1




    $begingroup$
    If you have to know complex analysis in order to do complex integration, then I can't do it. If complex integration is just normal integration but with a few $i$'s here and there, then I can do that.
    $endgroup$
    – clathratus
    Dec 18 '18 at 5:03
















3












$begingroup$

A Possible way: Consider
$$I(a)=int_{0}^{+infty}frac{arctan(ax)}{1+x^2+x^4} dx$$
and
$$I'(a)=int_{0}^{+infty}frac{x}{(1+x^2+x^4)(1+x^2a^2)}dx=int_{0}^{+infty}frac{1}{(1+y+y^2)(1+a^2y)}dx$$
and



$$frac{1}{(1+y+y^2)(1+a^2y)}= frac{-a^2y-a^2+1}{(a^4-a^2+1)(1+y+y^2)}+frac{a^4}{(a^4-a^2+1)(ay^2+1)}$$



we can also calculate $I'(a)$ by complex integration (if you've learned that).



Thanks to Dylan for his advice.






share|cite|improve this answer











$endgroup$













  • $begingroup$
    Instead of doing partial fractions wrt $x$, you might consider substituting $u=x^2$ first
    $endgroup$
    – Dylan
    Dec 18 '18 at 3:07












  • $begingroup$
    And it is still not so easy. I think there may be a more clever way to solve it.
    $endgroup$
    – Lau
    Dec 18 '18 at 3:15










  • $begingroup$
    I meant the first integration. $$frac12 int_0^infty frac{1}{(1+u+u^2)(1+a^2u)} du$$ and then do partial fractions?
    $endgroup$
    – Dylan
    Dec 18 '18 at 3:31












  • $begingroup$
    @Dylan You're right.
    $endgroup$
    – Lau
    Dec 18 '18 at 3:34








  • 1




    $begingroup$
    If you have to know complex analysis in order to do complex integration, then I can't do it. If complex integration is just normal integration but with a few $i$'s here and there, then I can do that.
    $endgroup$
    – clathratus
    Dec 18 '18 at 5:03














3












3








3





$begingroup$

A Possible way: Consider
$$I(a)=int_{0}^{+infty}frac{arctan(ax)}{1+x^2+x^4} dx$$
and
$$I'(a)=int_{0}^{+infty}frac{x}{(1+x^2+x^4)(1+x^2a^2)}dx=int_{0}^{+infty}frac{1}{(1+y+y^2)(1+a^2y)}dx$$
and



$$frac{1}{(1+y+y^2)(1+a^2y)}= frac{-a^2y-a^2+1}{(a^4-a^2+1)(1+y+y^2)}+frac{a^4}{(a^4-a^2+1)(ay^2+1)}$$



we can also calculate $I'(a)$ by complex integration (if you've learned that).



Thanks to Dylan for his advice.






share|cite|improve this answer











$endgroup$



A Possible way: Consider
$$I(a)=int_{0}^{+infty}frac{arctan(ax)}{1+x^2+x^4} dx$$
and
$$I'(a)=int_{0}^{+infty}frac{x}{(1+x^2+x^4)(1+x^2a^2)}dx=int_{0}^{+infty}frac{1}{(1+y+y^2)(1+a^2y)}dx$$
and



$$frac{1}{(1+y+y^2)(1+a^2y)}= frac{-a^2y-a^2+1}{(a^4-a^2+1)(1+y+y^2)}+frac{a^4}{(a^4-a^2+1)(ay^2+1)}$$



we can also calculate $I'(a)$ by complex integration (if you've learned that).



Thanks to Dylan for his advice.







share|cite|improve this answer














share|cite|improve this answer



share|cite|improve this answer








edited Dec 18 '18 at 3:38

























answered Dec 18 '18 at 2:57









LauLau

515315




515315












  • $begingroup$
    Instead of doing partial fractions wrt $x$, you might consider substituting $u=x^2$ first
    $endgroup$
    – Dylan
    Dec 18 '18 at 3:07












  • $begingroup$
    And it is still not so easy. I think there may be a more clever way to solve it.
    $endgroup$
    – Lau
    Dec 18 '18 at 3:15










  • $begingroup$
    I meant the first integration. $$frac12 int_0^infty frac{1}{(1+u+u^2)(1+a^2u)} du$$ and then do partial fractions?
    $endgroup$
    – Dylan
    Dec 18 '18 at 3:31












  • $begingroup$
    @Dylan You're right.
    $endgroup$
    – Lau
    Dec 18 '18 at 3:34








  • 1




    $begingroup$
    If you have to know complex analysis in order to do complex integration, then I can't do it. If complex integration is just normal integration but with a few $i$'s here and there, then I can do that.
    $endgroup$
    – clathratus
    Dec 18 '18 at 5:03


















  • $begingroup$
    Instead of doing partial fractions wrt $x$, you might consider substituting $u=x^2$ first
    $endgroup$
    – Dylan
    Dec 18 '18 at 3:07












  • $begingroup$
    And it is still not so easy. I think there may be a more clever way to solve it.
    $endgroup$
    – Lau
    Dec 18 '18 at 3:15










  • $begingroup$
    I meant the first integration. $$frac12 int_0^infty frac{1}{(1+u+u^2)(1+a^2u)} du$$ and then do partial fractions?
    $endgroup$
    – Dylan
    Dec 18 '18 at 3:31












  • $begingroup$
    @Dylan You're right.
    $endgroup$
    – Lau
    Dec 18 '18 at 3:34








  • 1




    $begingroup$
    If you have to know complex analysis in order to do complex integration, then I can't do it. If complex integration is just normal integration but with a few $i$'s here and there, then I can do that.
    $endgroup$
    – clathratus
    Dec 18 '18 at 5:03
















$begingroup$
Instead of doing partial fractions wrt $x$, you might consider substituting $u=x^2$ first
$endgroup$
– Dylan
Dec 18 '18 at 3:07






$begingroup$
Instead of doing partial fractions wrt $x$, you might consider substituting $u=x^2$ first
$endgroup$
– Dylan
Dec 18 '18 at 3:07














$begingroup$
And it is still not so easy. I think there may be a more clever way to solve it.
$endgroup$
– Lau
Dec 18 '18 at 3:15




$begingroup$
And it is still not so easy. I think there may be a more clever way to solve it.
$endgroup$
– Lau
Dec 18 '18 at 3:15












$begingroup$
I meant the first integration. $$frac12 int_0^infty frac{1}{(1+u+u^2)(1+a^2u)} du$$ and then do partial fractions?
$endgroup$
– Dylan
Dec 18 '18 at 3:31






$begingroup$
I meant the first integration. $$frac12 int_0^infty frac{1}{(1+u+u^2)(1+a^2u)} du$$ and then do partial fractions?
$endgroup$
– Dylan
Dec 18 '18 at 3:31














$begingroup$
@Dylan You're right.
$endgroup$
– Lau
Dec 18 '18 at 3:34






$begingroup$
@Dylan You're right.
$endgroup$
– Lau
Dec 18 '18 at 3:34






1




1




$begingroup$
If you have to know complex analysis in order to do complex integration, then I can't do it. If complex integration is just normal integration but with a few $i$'s here and there, then I can do that.
$endgroup$
– clathratus
Dec 18 '18 at 5:03




$begingroup$
If you have to know complex analysis in order to do complex integration, then I can't do it. If complex integration is just normal integration but with a few $i$'s here and there, then I can do that.
$endgroup$
– clathratus
Dec 18 '18 at 5:03











3












$begingroup$

Result



I find that the integral has a closed form given by



$$i = intlimits_0^{pi/2}logbigg(frac{2+sin2x}{2-sin2x}bigg)mathrm dx = frac{1}{3} left(8 C-pi log left(2+sqrt{3}right)right) simeq 1.06346tag{1}$$



where



$$C = sum _{k=1}^{infty } frac{(-1)^{k+1}}{(2 k-1)^2} simeq 0.915966$$



is Catalan's constant.



Heuristic derivation



Notice trivially that because of the symmtery of the integrand the integral can be written as twice the integral from $0$ to $frac{pi}{4}$ which we shall utilize in what follows.



The basic idea is the series expansion



$$log left(frac{1+z}{1-z}right)=2tanh ^{-1}(z) = 2 sum _{k=1}^{infty } frac{z^{2 k-1}}{2 k-1},|z|<1 tag{2}$$



The integral is then to be done over the odd powers of the $sin$ with the result



$$int_0^{frac{pi }{4}} sin ^{2 k-1}(2 x) , dx = frac{sqrt{pi } Gamma (k)}{4 Gamma left(k+frac{1}{2}right)}tag{3}$$



Assembling the pieces the sum to be taken to represent $i$ becomes



$$i_s = sum _{k=1}^{infty } frac{sqrt{pi } Gamma (k)}{(2 k-1) 2^{2 k-1} Gamma left(k+frac{1}{2}right)}tag{4}$$



and this sum is immediately computed by Mathematica to give the compact result $(1)$.



Let us make the sum more transparent using the chain



$$frac{sqrt{pi } Gamma (k)}{Gamma left(k+frac{1}{2}right)}=Bleft(frac{1}{2},kright)=int_0^1 frac{t^{k-1}}{sqrt{1-t}} , dttag{5}$$



and doing the sum under the integral



$$sum _{k=1}^{infty } frac{t^{k-1}}{(2 k-1) 2^{2 k-1}}=frac{tanh ^{-1}left(frac{sqrt{t}}{2}right)}{sqrt{t}}tag{6}$$



leads finally to the integral



$$int_0^1 frac{tanh ^{-1}left(frac{sqrt{t}}{2}right)}{sqrt{t} sqrt{1-t}} , dttag{7}$$



for which Mathematica again quickly gives (1).



But there must be a shorter way ... yes, it is, substituting $sin (2 x)=sqrt{t}$ in the original integral gives (7) directly.






share|cite|improve this answer











$endgroup$













  • $begingroup$
    I do truly work better with series. Thank you.
    $endgroup$
    – clathratus
    Dec 18 '18 at 18:22
















3












$begingroup$

Result



I find that the integral has a closed form given by



$$i = intlimits_0^{pi/2}logbigg(frac{2+sin2x}{2-sin2x}bigg)mathrm dx = frac{1}{3} left(8 C-pi log left(2+sqrt{3}right)right) simeq 1.06346tag{1}$$



where



$$C = sum _{k=1}^{infty } frac{(-1)^{k+1}}{(2 k-1)^2} simeq 0.915966$$



is Catalan's constant.



Heuristic derivation



Notice trivially that because of the symmtery of the integrand the integral can be written as twice the integral from $0$ to $frac{pi}{4}$ which we shall utilize in what follows.



The basic idea is the series expansion



$$log left(frac{1+z}{1-z}right)=2tanh ^{-1}(z) = 2 sum _{k=1}^{infty } frac{z^{2 k-1}}{2 k-1},|z|<1 tag{2}$$



The integral is then to be done over the odd powers of the $sin$ with the result



$$int_0^{frac{pi }{4}} sin ^{2 k-1}(2 x) , dx = frac{sqrt{pi } Gamma (k)}{4 Gamma left(k+frac{1}{2}right)}tag{3}$$



Assembling the pieces the sum to be taken to represent $i$ becomes



$$i_s = sum _{k=1}^{infty } frac{sqrt{pi } Gamma (k)}{(2 k-1) 2^{2 k-1} Gamma left(k+frac{1}{2}right)}tag{4}$$



and this sum is immediately computed by Mathematica to give the compact result $(1)$.



Let us make the sum more transparent using the chain



$$frac{sqrt{pi } Gamma (k)}{Gamma left(k+frac{1}{2}right)}=Bleft(frac{1}{2},kright)=int_0^1 frac{t^{k-1}}{sqrt{1-t}} , dttag{5}$$



and doing the sum under the integral



$$sum _{k=1}^{infty } frac{t^{k-1}}{(2 k-1) 2^{2 k-1}}=frac{tanh ^{-1}left(frac{sqrt{t}}{2}right)}{sqrt{t}}tag{6}$$



leads finally to the integral



$$int_0^1 frac{tanh ^{-1}left(frac{sqrt{t}}{2}right)}{sqrt{t} sqrt{1-t}} , dttag{7}$$



for which Mathematica again quickly gives (1).



But there must be a shorter way ... yes, it is, substituting $sin (2 x)=sqrt{t}$ in the original integral gives (7) directly.






share|cite|improve this answer











$endgroup$













  • $begingroup$
    I do truly work better with series. Thank you.
    $endgroup$
    – clathratus
    Dec 18 '18 at 18:22














3












3








3





$begingroup$

Result



I find that the integral has a closed form given by



$$i = intlimits_0^{pi/2}logbigg(frac{2+sin2x}{2-sin2x}bigg)mathrm dx = frac{1}{3} left(8 C-pi log left(2+sqrt{3}right)right) simeq 1.06346tag{1}$$



where



$$C = sum _{k=1}^{infty } frac{(-1)^{k+1}}{(2 k-1)^2} simeq 0.915966$$



is Catalan's constant.



Heuristic derivation



Notice trivially that because of the symmtery of the integrand the integral can be written as twice the integral from $0$ to $frac{pi}{4}$ which we shall utilize in what follows.



The basic idea is the series expansion



$$log left(frac{1+z}{1-z}right)=2tanh ^{-1}(z) = 2 sum _{k=1}^{infty } frac{z^{2 k-1}}{2 k-1},|z|<1 tag{2}$$



The integral is then to be done over the odd powers of the $sin$ with the result



$$int_0^{frac{pi }{4}} sin ^{2 k-1}(2 x) , dx = frac{sqrt{pi } Gamma (k)}{4 Gamma left(k+frac{1}{2}right)}tag{3}$$



Assembling the pieces the sum to be taken to represent $i$ becomes



$$i_s = sum _{k=1}^{infty } frac{sqrt{pi } Gamma (k)}{(2 k-1) 2^{2 k-1} Gamma left(k+frac{1}{2}right)}tag{4}$$



and this sum is immediately computed by Mathematica to give the compact result $(1)$.



Let us make the sum more transparent using the chain



$$frac{sqrt{pi } Gamma (k)}{Gamma left(k+frac{1}{2}right)}=Bleft(frac{1}{2},kright)=int_0^1 frac{t^{k-1}}{sqrt{1-t}} , dttag{5}$$



and doing the sum under the integral



$$sum _{k=1}^{infty } frac{t^{k-1}}{(2 k-1) 2^{2 k-1}}=frac{tanh ^{-1}left(frac{sqrt{t}}{2}right)}{sqrt{t}}tag{6}$$



leads finally to the integral



$$int_0^1 frac{tanh ^{-1}left(frac{sqrt{t}}{2}right)}{sqrt{t} sqrt{1-t}} , dttag{7}$$



for which Mathematica again quickly gives (1).



But there must be a shorter way ... yes, it is, substituting $sin (2 x)=sqrt{t}$ in the original integral gives (7) directly.






share|cite|improve this answer











$endgroup$



Result



I find that the integral has a closed form given by



$$i = intlimits_0^{pi/2}logbigg(frac{2+sin2x}{2-sin2x}bigg)mathrm dx = frac{1}{3} left(8 C-pi log left(2+sqrt{3}right)right) simeq 1.06346tag{1}$$



where



$$C = sum _{k=1}^{infty } frac{(-1)^{k+1}}{(2 k-1)^2} simeq 0.915966$$



is Catalan's constant.



Heuristic derivation



Notice trivially that because of the symmtery of the integrand the integral can be written as twice the integral from $0$ to $frac{pi}{4}$ which we shall utilize in what follows.



The basic idea is the series expansion



$$log left(frac{1+z}{1-z}right)=2tanh ^{-1}(z) = 2 sum _{k=1}^{infty } frac{z^{2 k-1}}{2 k-1},|z|<1 tag{2}$$



The integral is then to be done over the odd powers of the $sin$ with the result



$$int_0^{frac{pi }{4}} sin ^{2 k-1}(2 x) , dx = frac{sqrt{pi } Gamma (k)}{4 Gamma left(k+frac{1}{2}right)}tag{3}$$



Assembling the pieces the sum to be taken to represent $i$ becomes



$$i_s = sum _{k=1}^{infty } frac{sqrt{pi } Gamma (k)}{(2 k-1) 2^{2 k-1} Gamma left(k+frac{1}{2}right)}tag{4}$$



and this sum is immediately computed by Mathematica to give the compact result $(1)$.



Let us make the sum more transparent using the chain



$$frac{sqrt{pi } Gamma (k)}{Gamma left(k+frac{1}{2}right)}=Bleft(frac{1}{2},kright)=int_0^1 frac{t^{k-1}}{sqrt{1-t}} , dttag{5}$$



and doing the sum under the integral



$$sum _{k=1}^{infty } frac{t^{k-1}}{(2 k-1) 2^{2 k-1}}=frac{tanh ^{-1}left(frac{sqrt{t}}{2}right)}{sqrt{t}}tag{6}$$



leads finally to the integral



$$int_0^1 frac{tanh ^{-1}left(frac{sqrt{t}}{2}right)}{sqrt{t} sqrt{1-t}} , dttag{7}$$



for which Mathematica again quickly gives (1).



But there must be a shorter way ... yes, it is, substituting $sin (2 x)=sqrt{t}$ in the original integral gives (7) directly.







share|cite|improve this answer














share|cite|improve this answer



share|cite|improve this answer








edited Dec 18 '18 at 15:10

























answered Dec 18 '18 at 13:37









Dr. Wolfgang HintzeDr. Wolfgang Hintze

3,735620




3,735620












  • $begingroup$
    I do truly work better with series. Thank you.
    $endgroup$
    – clathratus
    Dec 18 '18 at 18:22


















  • $begingroup$
    I do truly work better with series. Thank you.
    $endgroup$
    – clathratus
    Dec 18 '18 at 18:22
















$begingroup$
I do truly work better with series. Thank you.
$endgroup$
– clathratus
Dec 18 '18 at 18:22




$begingroup$
I do truly work better with series. Thank you.
$endgroup$
– clathratus
Dec 18 '18 at 18:22


















draft saved

draft discarded




















































Thanks for contributing an answer to Mathematics Stack Exchange!


  • Please be sure to answer the question. Provide details and share your research!

But avoid



  • Asking for help, clarification, or responding to other answers.

  • Making statements based on opinion; back them up with references or personal experience.


Use MathJax to format equations. MathJax reference.


To learn more, see our tips on writing great answers.




draft saved


draft discarded














StackExchange.ready(
function () {
StackExchange.openid.initPostLogin('.new-post-login', 'https%3a%2f%2fmath.stackexchange.com%2fquestions%2f3044687%2fbaaad-int-0-pi-2-log-bigg-frac2-sin2x2-sin2x-bigg-mathrm-dx%23new-answer', 'question_page');
}
);

Post as a guest















Required, but never shown





















































Required, but never shown














Required, but never shown












Required, but never shown







Required, but never shown

































Required, but never shown














Required, but never shown












Required, but never shown







Required, but never shown







Popular posts from this blog

Le Mesnil-Réaume

Bundesstraße 106

Ida-Boy-Ed-Garten